¿Cómo se resuelven estas operaciones de Álgebra?

6) Halla a positivo y más pequeño posible tal que:

  1. a) 1234 = a (5)
  2. b) 2240 = a (3)

NOTA: En el ejercicio en lugar de a viene el símbolo de infinito pero no lo pude insertar

1 Respuesta

Respuesta
1

·

·

¡Hola Miguel Angel!

No entiendo nada, no sé si es uno o dos ejercicios y del resto nada. Mira a ver si puedes tomar una imagen de la pantalla y la pones aquí con el icono "añadir imagen" que está a la izquierda, donde sale un cuadro de paisaje. A lo mejor deberías explicar con más palabras todo, porque la notación depende del tema al que nos estemos rediriendo.
Para tomar la imagen puedes usar el programa "Recortes" si estás en Windows 7, 8, 10.

Saludos.

:

:

Si yo tampoco entiendo Profesor, adjunto una imagen, son eso dos ejercicios, muchas gracias por su tiempo y su ayuda

·

·

¡Hola!

Si quieres el 5 lo entiendo

A los factores primos de exponente impar de 9360 hay que quitarles un ecponente de modo qe queden todos pares y entonces será un cuadrado en N

Luego descomponemos 9360 en factores primos

9360 | 2

4680 | 2

2340 | 2

1170 | 2

  585 | 3

  195 | 3

    65 | 5

    13 | 13

1

Luego la descomposición es:

9360 = 2^4 · 3^2 · 5 · 13

El 5 y el 13 tienen exponente 1, luego tendremos que dividir entre ellos para que quede un cuadrado en N.

Poe ello el menor entero por el que hay que dividir es

5·13 = 65

·

En el segundo no estoy seguro lo que quieren decir pero supongo que es congruente con alfa módulo 5. Es que yo eso siempre lo he visto escrito

$$\begin{align}&1)\quad 1234 \equiv \alpha\, (mod \,5)\\&\\&\text{supongo que será eso lo que piden, si no ya me lo dirás}\\&1234\, \equiv 4\, (mod\, 5)\\&\text{ya que 4 es el resto de la división entera 1234/5}\\&\\&\\&2)\quad 2^{240} \equiv  \alpha \,(mod\, 3)\\&\\&\text{Haya que hallar el resto de }2^{240}/3\\&\text {Vamos probando}\\&2^1 = 2 \equiv 2\;(mod\, 3)\\&2^2= 4\equiv1\; (mod\, 3)\\&\\&\text {Entonces}\\&\\&2^{240} = (2^2)^{120}\\&\\&\text{Como}\\&2^2\equiv 1\;(mod\, 3)\\&entonces\\&\\&(2^2)^{120}\equiv 1^{120} (mod\,3)\\&\\&2^{240}\equiv 1\;(mod \,3)\\&\\&\text{Luego }\alpha=1\end{align}$$

Y eso es todo, sa lu dos.

:

.

¡Gracias! Profesor me parece que es así como me lo está planteando, de verdad me ha ayudado muchísimo además como lo expone ahora me hace idea, muchas gracias

Añade tu respuesta

Haz clic para o

Más respuestas relacionadas